Question

1. A firm uses labor and machines to produce output according to the production function f(L, M) 2L2 M , where L is the numbe
0 0
Add a comment Improve this question Transcribed image text
Answer #1

1 - q = 2 M ; w= $20 ; g = $5 can least cost combination is determined where MRTS = 0. - MRT5 MB 2(1) M MPM aglam 264), + (z

Add a comment
Know the answer?
Add Answer to:
1. A firm uses labor and machines to produce output according to the production function f(L,...
Your Answer:

Post as a guest

Your Name:

What's your source?

Earn Coins

Coins can be redeemed for fabulous gifts.

Not the answer you're looking for? Ask your own homework help question. Our experts will answer your question WITHIN MINUTES for Free.
Similar Homework Help Questions
  • Question 7 rding to the production function: uses labor and machines to produce output according to...

    Question 7 rding to the production function: uses labor and machines to produce output according to the where Lis ALK) = 41/212, ere is the number of units of labor used and K is the amount of capita or is $40 per unit and the cost of employing capital is $10 per unit. mount of capital employed. The cost (0): On the graph below, draw an isocost line for this firm that includes combin capital and labor that cost $400...

  • A firm uses labour and capital to produce output according to the production function ??(??, ??) = 4??0.5??0.5, where L...

    A firm uses labour and capital to produce output according to the production function ??(??, ??) = 4??0.5??0.5, where L is the number of units of labour and K is the units of capital. The cost of labour is $40 a unit and the cost of capital is $10 a unit. a) On a graph, draw an isocost line for this firm, showing combinations of capital and labour that cost $400 and another isocost line showing combinations that cost $200....

  • A firm uses labor L to produce output Q, using a production function Q = 2L....

    A firm uses labor L to produce output Q, using a production function Q = 2L. The cost of L is $10 per unit. If the firm needs to produce exactly 200 units of Q, what will this cost?

  • 11. Consider the production function: f(K,L)=K+L. Let w and r denote the price of labor and...

    11. Consider the production function: f(K,L)=K+L. Let w and r denote the price of labor and capital, and let p denote the price of the output good. (a) Find the cost minimizing input bundle and the cost function. (b) Find the profit maximizing output level and the profit function. 12. Consider a firm with production function f(K,L) = K +L. (a) Suppose that capital level is currently fixed at K = 10. Find the short term production cost function for...

  • 10. Consider the production function: f(KL)=K L. Let wandr denote the price of labor and capital,...

    10. Consider the production function: f(KL)=K L. Let wandr denote the price of labor and capital, and let p denote the price of the output good. (a) Find the cost minimizing input bundle and the cost function as a function of w., and q. (b) Find the profit maximizing output level and the profit as a function of w, r, and p. 11. Consider the production function: f(KL)=K+L. Let w and r denote the price of labor and capital, and...

  • A firm’s production technology is given by the production function q = 0.25 LK where L...

    A firm’s production technology is given by the production function q = 0.25 LK where L represents labor hours, K machine hours and q the amount of output. The market wage and rental rates are, w= $16 and r = $256. The firm is operating in the long run where it can adjust both inputs. (a) Suppose that the firm currently is using ten labor hours for each machine hour. Is it minimizing its long run total cost? If so...

  • A firm produces output according to the production function: Q = F(K,L) = 2K + 2L....

    A firm produces output according to the production function: Q = F(K,L) = 2K + 2L. a. How much output is produced when K = 2 and L = 3? b. If the wage rate is $65 per hour and the rental rate on capital is $35 per hour, what is the cost-minimizing input mix for producing 4 units of output? Capital: Labor:

  • 9. Suppose the firm's production function is given by f(K,L) = min (Kº,L"} (a) For what values of a will the firm e...

    9. Suppose the firm's production function is given by f(K,L) = min (Kº,L"} (a) For what values of a will the firm exhibit decreasing returns to scale? Constant returns to scale? Increasing returns to scale? (b) Derive the long-run cost function and the optimal input choices. (c) Suppose the capital is fixed at K = 10,000 and a = 1. Assuming that the firm wants to produce less than 100 units, derive 10. Consider the production function: f(K,L)=KLI. Let w...

  • - Julia operates a cost-minimizing firm that produces a single output using labor (L) and capital...

    - Julia operates a cost-minimizing firm that produces a single output using labor (L) and capital (K). The firm's production function is Q f(L, K) = min{L, K}}. The per-unit price of labor is w = 1 and the per-unit price of capital is r = 1. Recently, the government imposed a tax on Julia's firm: For each unit of labor that Julia employs, she must pay a tax of £t to the government. (a) Graph the Q unit of...

  • 9. Suppose the firm's production function is given by f(K,L) min (K",L" (a) For what values...

    9. Suppose the firm's production function is given by f(K,L) min (K",L" (a) For what values of a will the firm exhibit decreasing returns to scale? Constant returns to scale? Increasing returns to scale? (b) Derive the long-run cost function and the optimal input choices. (c) Suppose the capital is fixed at R = 10,000 and a =. Assuming that the firm wants to produce less than 100 units, derive 10. Consider the production function: f(K, L) = KLi. Let...

ADVERTISEMENT
Free Homework Help App
Download From Google Play
Scan Your Homework
to Get Instant Free Answers
Need Online Homework Help?
Ask a Question
Get Answers For Free
Most questions answered within 3 hours.
ADVERTISEMENT
ADVERTISEMENT
ADVERTISEMENT